$$\newcommand{\mtn}{\mathbb{N}}\newcommand{\mtns}{\mathbb{N}^*}\newcommand{\mtz}{\mathbb{Z}}\newcommand{\mtr}{\mathbb{R}}\newcommand{\mtk}{\mathbb{K}}\newcommand{\mtq}{\mathbb{Q}}\newcommand{\mtc}{\mathbb{C}}\newcommand{\mch}{\mathcal{H}}\newcommand{\mcp}{\mathcal{P}}\newcommand{\mcb}{\mathcal{B}}\newcommand{\mcl}{\mathcal{L}} \newcommand{\mcm}{\mathcal{M}}\newcommand{\mcc}{\mathcal{C}} \newcommand{\mcmn}{\mathcal{M}}\newcommand{\mcmnr}{\mathcal{M}_n(\mtr)} \newcommand{\mcmnk}{\mathcal{M}_n(\mtk)}\newcommand{\mcsn}{\mathcal{S}_n} \newcommand{\mcs}{\mathcal{S}}\newcommand{\mcd}{\mathcal{D}} \newcommand{\mcsns}{\mathcal{S}_n^{++}}\newcommand{\glnk}{GL_n(\mtk)} \newcommand{\mnr}{\mathcal{M}_n(\mtr)}\DeclareMathOperator{\ch}{ch} \DeclareMathOperator{\sh}{sh}\DeclareMathOperator{\th}{th} \DeclareMathOperator{\vect}{vect}\DeclareMathOperator{\card}{card} \DeclareMathOperator{\comat}{comat}\DeclareMathOperator{\imv}{Im} \DeclareMathOperator{\rang}{rg}\DeclareMathOperator{\Fr}{Fr} \DeclareMathOperator{\diam}{diam}\DeclareMathOperator{\supp}{supp} \newcommand{\veps}{\varepsilon}\newcommand{\mcu}{\mathcal{U}} \newcommand{\mcun}{\mcu_n}\newcommand{\dis}{\displaystyle} \newcommand{\croouv}{[\![}\newcommand{\crofer}{]\!]} \newcommand{\rab}{\mathcal{R}(a,b)}\newcommand{\pss}[2]{\langle #1,#2\rangle} $$
Bibm@th

Math spé : Exercices sur les intégrales à paramètres

Permutation suites et intégrales
Exercice 1 - Théorème de convergence dominée - 1 [Signaler une erreur] [Ajouter à ma feuille d'exos]
Enoncé
Déterminer la limite, lorsque $n$ tend vers $+\infty$, des suites suivantes : $$\begin{array}{lll} \displaystyle \mathbf 1.\ \left(\int_0^{\pi/4}(\tan t)^n dt\right)&\quad\quad&\mathbf 2. \displaystyle\left(\int_1^{+\infty}\frac{dt}{1+t^n}\right)\\ \mathbf 3. \displaystyle\left(\int_0^{+\infty}\frac{e^{-x/n}}{1+x^2}dx\right)&\quad\quad& \mathbf 4. \displaystyle\left(\int_0^1 f(t^n)dt\right),\ f:[0,1]\to\mathbb R\textrm{ continue}. \end{array}$$
Indication
Corrigé
Exercice 2 - Majoration un peu subtile [Signaler une erreur] [Ajouter à ma feuille d'exos]
Enoncé
Pour $n\geq 1$ et $x\in [0,1]$, on pose $f_n(x)=nx(1-x)^n$.
  1. Démontrer que, pour tout $x\in [0,1]$ et tout $n\geq 1$, on a $|f_n(x)|\leq 1$.
  2. En déduire $\lim_{n\to+\infty}\int_0^1 nx(1-x)^n dx$.
Indication
Corrigé
Enoncé
Déterminer la limite des suites suivantes : $$\begin{array}{lll} \mathbf{1.}\ \left(\int_0^{+\infty}\frac{dx}{x^n+e^x}\right)&\quad\quad&\mathbf{2.}\ \left(\int_0^{+\infty}\frac{dx}{(1+x^2)\sqrt[n]{1+x^n}}\right) \end{array}$$
Indication
Corrigé
Enoncé
Déterminer la limite des suites suivantes : $$\begin{array}{lll} \mathbf{1.}\quad \left(\int_0^{+\infty}\arctan(nx)e^{-x^n}dx\right)&\quad\quad& \mathbf{2.}\quad \left(\int_0^{+\infty}\frac{x^n}{1+x^{n+2}}dx\right) \end{array}$$
Indication
Corrigé
Exercice 5 - Avec les bornes qui varient [Signaler une erreur] [Ajouter à ma feuille d'exos]
Enoncé
Déterminer la limite, quand $n\to+\infty$, de $$\int_0^n \left(1-\frac{x}{n}\right)^n dx.$$
Indication
Corrigé
Exercice 6 - Après un changement de variables [Signaler une erreur] [Ajouter à ma feuille d'exos]
Enoncé
Soit $f:[0,+\infty[\to\mathbb R$ continue.
  1. On suppose que $f$ admet une limite $\ell$ en $+\infty$. Déterminer $$\lim_{n\to+\infty}\frac 1n\int_0^n f(t)dt.$$
  2. On suppose que $f$ est bornée et que $f\geq 0$. Déterminer $$\lim_{n\to+\infty}\int_0^{+\infty}nf(t)e^{-nt}dt.$$
Indication
Corrigé
Enoncé
Soit la fonction $\Gamma$ définie pour $x>0$ par $$\Gamma(x)=\int_0^{+\infty}t^{x-1}e^{-t}dt.$$ En introduisant $I_n(x)=\int_0^n \left(1-\frac{t}{n}\right)^n t^{x-1}dt$, démontrer que $$\Gamma(x)=\lim_{n\to+\infty}\frac{n! n^x}{x(x+1)\dots(x+n)}.$$
Indication
Corrigé
Enoncé
Soit $]a,b[$ un intervalle non-vide de $\mathbb R$. Soient $(a_n)$ et $(b_n)$ deux suites de nombres réels. On suppose que pour tout $x\in]a,b[$, $$\lim_{n\to+\infty}\big(a_n \sin(nx)+b_n\cos(nx)\big)=0.$$ On souhaite prouver que $(a_n)$ et $(b_n)$ convergent vers $0$. On raisonne par l'absurde et on suppose que ce n'est pas le cas.
  1. Démontrer qu'il existe une suite strictement croissante d'entiers $(n_k)$ telle que, pour tout $x\in]a,b[$, la suite $$f_k(x)=\frac{\big(a_{n_k}\sin(n_kx)+b_{n_k}\cos(n_kx)\big)^2}{a_{n_k}^2+b_{n_k}^2}$$ tende vers 0 pour tout $x\in]a,b[$.
  2. En calculant de deux façons différentes $\lim_{k\to+\infty}\int_a^b f_k(x)dx$, trouver une contradiction.
Indication
Corrigé
Exercice 9 - Changement de variables.... [Signaler une erreur] [Ajouter à ma feuille d'exos]
Enoncé
Soit $(a_n)$ une suite de $]0,+\infty[$ qui converge vers 0. Soit $f:\mathbb R^+\to\mathbb R$ continue et bornée. Déterminer la limite de $\displaystyle \int_0^{+\infty}\frac{a_nf(x)}{a_n^2+x^2}dx$.
Indication
Corrigé
Équivalent et développement asymptotique d'intégrales à paramètres
Enoncé
Pour $n\geq 1,$ on pose $\displaystyle I_n=\int_1^{+\infty}\frac{dx}{nx^2+\sqrt x}$. Déterminer un équivalent simple de $I_n$ lorsque $n$ tend vers $+\infty$.
Indication
Corrigé
Exercice 11 - Après un changement de variables [Signaler une erreur] [Ajouter à ma feuille d'exos]
Enoncé
Démontrer l'égalité suivante : $$\lim_{n\to+\infty} n\int_1^{+\infty}e^{-x^n}dx=\int_1^{+\infty}\frac{e^{-x}}xdx.$$
Indication
Corrigé
Enoncé
Soit $f:\mathbb R_+\to\mathbb R$ une fonction continue et bornée.
  1. On pose $I_n=n\int_0^{+\infty}f(x)e^{-nx}dx$. Déterminer $\ell=\lim_{n\to+\infty}I_n$.
  2. On suppose de plus que $f$ est $\mathcal C^1$, de dérivée bornée, et vérifie $f'(0)\neq 0$. Déterminer un équivalent de $\ell-I_n$.
  3. On ne suppose plus que $f$ est $\mathcal C^1$, mais uniquement que $f$ est dérivable en $0$ avec $f'(0)\neq 0$. On définit $g$ sur $\mathbb R_+$ par $g(t)=\frac{f(t)-f(0)}{t}$ si $t>0$ et $g(0)=f'(0)$.
    1. Justifier que $g$ est bornée.
    2. Démontrer que le résultat obtenu à la question précédente reste vrai.
Indication
Corrigé
Enoncé
On pose, pour $n\geq 1$, $$I_n=\int_0^1 \frac{1}{1+t^n}dt.$$
  1. Déterminer $\ell=\lim_{n\to+\infty}I_n$.
  2. Déterminer un équivalent de $\ell-I_n$.
Indication
Corrigé
Exercice 14 - Un développement asymptotique [Signaler une erreur] [Ajouter à ma feuille d'exos]
Enoncé
Donner un développement asymptotique à deux termes de $\displaystyle I_n=\int_0^{+\infty}\frac{e^{-x}}{x+n}dx.$
Indication
Corrigé
Permutation séries et intégrales
Exercice 15 - Deux versions de la permutation séries/intégrales [Signaler une erreur] [Ajouter à ma feuille d'exos]
Enoncé
  1. Démontrer que $\displaystyle \int_0^1\frac{\ln(t)}{t-1}dt=\sum_{n=0}^{+\infty}\frac 1{(n+1)^2}.$
  2. Démontrer que $\displaystyle \int_0^1\frac{\ln(t)}{t+1}dt=\sum_{n=0}^{+\infty}\frac {(-1)^{n+1}}{(n+1)^2}.$
Indication
Corrigé
Enoncé
    1. Démontrer que $$\sum_{n=0}^{+\infty}\int_0^1 x^{2n}(1-x)dx=\int_0^1\frac{dx}{1+x}.$$
    2. En déduire que $$\sum_{k=1}^{+\infty}\frac{(-1)^{k+1}}{k}=\ln 2.$$
  1. En calculant de deux façons $\sum_{n=0}^{+\infty}(-1)^n\int_0^1 x^{2n}(1-x)dx$, déterminer la valeur de la somme $$\sum_{n=0}^{+\infty}\frac{(-1)^n}{(2n+1)(2n+2)}.$$
Indication
Corrigé
Exercice 17 - Des limites du théorème d'intégration terme à terme [Signaler une erreur] [Ajouter à ma feuille d'exos]
Enoncé
On souhaite dans cet exercice démontrer l'égalité suivante : $$\int_1^{+\infty}\frac1{1+t^3}dt=\sum_{n=0}^{+\infty}\frac{(-1)^n}{3n+2}.$$ Pour cela, on veut partir de l'égalité $$\frac1{1+t^3}=\sum_{n=0}^{+\infty}\frac{(-1)^n}{t^{3n+3}}$$ valide pour $t>1$.
  1. Expliquer pourquoi on ne peut pas appliquer le théorème d'intégration terme à terme.
  2. Pour $n\geq 0$ et $t>1,$ on pose $R_n(t)=\sum_{k=n+1}^{+\infty}\frac{(-1)^n}{t^{3n+3}}.$ Démontrer que $\int_1^{+\infty}R_n(t)dt$ tend vers $0$, et conclure.
Indication
Corrigé
Enoncé
Soit $(a_n)$ une suite de nombres complexes telle que $\sum_n a_n n!$ converge absolument. Démontrer que $$\int_0^{+\infty}\left(e^{-x}\sum_{n=0}^{+\infty}a_nx^n\right)dx=\sum_{n=0}^{+\infty}a_n n!.$$
Indication
Corrigé
Enoncé
  1. Démontrer que $\int_0^{+\infty}\frac{t}{e^t-1}dt=\sum_{n\geq 1}\frac 1{n^2}$.
  2. Plus généralement, démontrer que, pour tous $a,b>0$, on a $$\int_0^{+\infty}\frac{te^{-at}}{1-e^{-bt}}dt=\sum_{n=0}^{+\infty}\frac 1{(a+bn)^2}.$$
Indication
Corrigé
Exercice 20 - Après un développement en série entière [Signaler une erreur] [Ajouter à ma feuille d'exos]
Enoncé
Démontrer que $\int_0^{+\infty}\cos(\sqrt x)e^{-x}dx=\sum_{n=0}^{+\infty}(-1)^n \frac{n!}{(2n)!}$.
Indication
Corrigé
Exercice 21 - Egalité série/intégrale [Signaler une erreur] [Ajouter à ma feuille d'exos]
Enoncé
Démontrer que $\int_0^1 \ln(x)\ln(1-x)dx=\sum_{n\geq 1}\frac{1}{n(n+1)^2}.$
Indication
Corrigé
Exercice 22 - Sans le théorème d'intégration terme à terme [Signaler une erreur] [Ajouter à ma feuille d'exos]
Enoncé
Soient $a$ et $b$ deux réels strictement positifs.
  1. Pour $t\in ]0,1[$, écrire $\frac{t^{a-1}}{1+t^b}$ comme somme d'une série $\sum_{n\geq 0}u_n(t)$.
  2. Déterminer la nature de la série $\sum_{n\geq 0}\int_0^1 |u_n(t)|dt$. Que peut-on en déduire?
  3. On pose $S_N(t)=\sum_{n=0}^N u_n(t)$. Démontrer que $$\int_0^1 \frac{t^{a-1}}{1+t^b}dt=\lim_{N\to+\infty}\int_0^1 S_N(t)dt.$$
  4. En déduire que $$\sum_{n=0}^{+\infty}\frac{(-1)^n}{a+nb}=\int_0^1 \frac{t^{a-1}}{1+t^b}dt$$ puis la valeur de $\sum_{n\geq 0}\frac{(-1)^n}{3n+1}$.
Indication
Corrigé
Exercice 23 - Somme trigonométrique [Signaler une erreur] [Ajouter à ma feuille d'exos]
Enoncé
Soit $\theta$ un réel non congru à $0$ modulo $2\pi$.
  1. Démontrer que $$\Re e\left(\int_0^1 \frac{e^{i\theta}}{1-e^{i\theta}x}dx\right)=-\ln\left|2\sin\frac{\theta}2\right|.$$
  2. Démontrer que $$\sum_{n=0}^{+\infty}\int_0^1e^{i(n+1)\theta}x^n=\int_0^1 \frac{e^{i\theta}}{1-e^{i\theta}x}dx.$$
  3. Conclure que $$\sum_{n\geq 1}\frac{\cos(n\theta)}{n}=-\ln\left|2\sin\frac{\theta}{2}\right|.$$
Indication
Corrigé
Enoncé
Le but de cet exercice est de démontrer la remarquable identité : $$\int_0^1\frac{dx}{x^x}=\sum_{m=1}^{+\infty}\frac1{m^m}.$$
  1. Justifier la convergence de chacun des membres de l'égalité précédente.
  2. Pour $p$ et $q$ des entiers naturels, on pose $I(p,q)=\int_0^1 x^p(\ln x)^qdx$; justifier la convergence de cette intégrale.
  3. Calculer $I(m,0)$ pour $m\in\mathbb N$.
  4. En déduire la valeur de $I(p,q)$ pour tout couple $(p,q)\in\mathbb N^2$.
  5. En développant $\frac{1}{x^x}$ en série, justifier que $$\int_0^1\frac{dx}{x^x}=\sum_{n=0}^{+\infty}\frac{(-1)^n}{n!}I(n,n).$$
  6. Conclure.
Indication
Corrigé
Etude de fonctions définies par une intégrale
Exercice 25 - Continuité d'une intégrale à paramètres [Signaler une erreur] [Ajouter à ma feuille d'exos]
Enoncé
  1. Démontrer que $\displaystyle F:x\mapsto \int_0^{+\infty}\frac{\sin(x^2t^2)}{1+t^2}dt$ est définie et continue sur $\mathbb R$.
  2. Démontrer que $\displaystyle G:x\mapsto \int_0^{+\infty}\sin(x^2t^2)e^{-xt}dt$ est définie et continue sur $]0,+\infty[$.
Indication
Corrigé
Exercice 26 - Une intégrale à paramètres de classe $\mathcal C^\infty$ [Signaler une erreur] [Ajouter à ma feuille d'exos]
Enoncé
  1. Déterminer l'ensemble de définition $D$ de $\displaystyle x\mapsto \int_0^{+\infty}\frac{e^{-tx}}{1+t^2}dt.$
  2. Montrer que $F$ est de classe $\mathcal C^\infty$ sur $D\backslash\{0\}$.
Indication
Corrigé
Exercice 27 - Calcul d'une intégrale impropre par dérivation [Signaler une erreur] [Ajouter à ma feuille d'exos]
Enoncé
On pose, pour $x\in\mathbb R$, $$F(x)=\int_0^{+\infty}\frac{\sin(xt)}te^{-t}dt.$$
  1. Justifier que $F$ est bien définie sur $\mathbb R$.
  2. Justifier que $F$ est $\mathcal C^1$ et donner une expression de $F'(x)$ pour tout $x\in\mathbb R$.
  3. Calculer $F'(x)$.
  4. En déduire une expression simplifiée de $F(x)$.
Indication
Corrigé
Enoncé
Pour $n\geq 1$ et $x>0$, on pose $$I_n(x)=\int_0^{+\infty}\frac{dt}{(x^2+t^2)^n}.$$
  1. Justifier l'existence de $I_n(x)$.
  2. Calculer $I_1(x)$.
  3. Démontrer que $I_n$ est de classe $C^1$ sur $]0,+\infty[$ et former une relation entre $I'_n(x)$ et $I_{n+1}(x)$.
  4. En déduire qu'il existe une suite $(\lambda_n)$ telle que, pour tout $x>0$, on a $$I_n(x)=\frac{\lambda_n}{x^{2n-1}}.$$ Que vaut $\lambda_n$?
Indication
Corrigé
Exercice 29 - Solution d'une équation différentielle [Signaler une erreur] [Ajouter à ma feuille d'exos]
Enoncé
On pose $F(x)=\int_0^{+\infty}\frac{e^{-xt}}{1+t^2}dt$.
  1. Démontrer que $F$ est définie sur $]0,+\infty[$.
  2. Justifier que $F$ tend vers $0$ en $+\infty$.
  3. Démontrer que $F$ est solution sur $]0,+\infty[$ de l'équation $y''+y=\frac 1x$.
Indication
Corrigé
Enoncé
Pour $x>0$, on définit $$f(x)=\int_0^{\pi/2}\frac{\cos(t)}{t+x}dt.$$
  1. Justifier que $f$ est de classe $\mathcal C^1$ sur $]0,+\infty[$, et étudier les variations de $f$.
  2. Déterminer la limite de $f$ en $+\infty$.
  3. En utilisant $1-\frac {t^2}2\leq \cos t\leq 1$, valable pour $t\in[0,\pi/2]$, démontrer que $$f(x)\sim_{0^+}-\ln x.$$
  4. Déterminer un équivalent de $f$ en $+\infty$.
Indication
Corrigé
Enoncé
Soient $a,b>0$. On définit, pour $x\in\mathbb R$, $$F(x)=\int_0^{+\infty}\frac{e^{-at}-e^{-bt}}t\cos(xt)dt.$$
  1. Justifier l'existence de $F(x)$.
  2. Prouver que $F$ est $C^1$ sur $\mathbb R$ et calculer $F'(x)$.
  3. En déduire qu'il existe une constante $C\in\mathbb R$ telle que, pour tout $x\in\mathbb R$, $$F(x)=\frac 12\ln\left(\frac{b^2+x^2}{a^2+x^2}\right)+C.$$
  4. Justifier que, pour tout $x\in\mathbb R^*$, on a $$F(x)=-\frac1x\int_0^{+\infty}\psi'(t)\sin(xt)dt,$$ où $\psi(t)=\frac{e^{-at}-e^{-bt}}t$.
  5. En déduire la valeur de $C$.
Indication
Corrigé
Exercice 32 - Une fonction définie par une intégrale [Signaler une erreur] [Ajouter à ma feuille d'exos]
Enoncé
Pour $x\in\mathbb R$, on pose $$\gamma(x)=\int_0^{+\infty}\frac{\cos(2tx)}{\cosh^2(t)}dt.$$
  1. Justifier que $\gamma$ est définie sur $\mathbb R$.
  2. Démontrer que $\gamma$ est continue sur $\mathbb R$.
  3. Etablir la relation suivante : pour tout $x\in\mathbb R$, \[ \gamma(x)=1-4x\int_0^{+\infty}\frac{\sin(2xt)}{1+e^{2t}}dt. \]
  4. En déduire que, pour tout $x\in\mathbb R$, \[ \gamma(x)=1+2x^2\sum_{k=1}^{+\infty}\frac{(-1)^k}{k^2+x^2}. \]
Indication
Corrigé
Enoncé
On pose $$F(x)=\int_0^{+\infty}\frac{dt}{1+t^x}.$$
  1. Déterminer le domaine de définition de $F$ et démontrer que $F$ est continue sur ce domaine de définition.
  2. Démontrer que $F$ est de classe $\mathcal C^1$ sur $]1,+\infty[$ et démontrer que, pour tout $x>1$, $$F'(x)=\int_1^{+\infty}\frac{t^x\ln (t)}{(1+t^x)^2}\left(\frac 1{t^2}-1\right)dt.$$ En déduire le sens de variation de $F$.
  3. Déterminer la limite de $F$ en $+\infty$.
  4. On suppose que $F$ admet une limite $\ell$ en $1^+$. Démontrer que pour tout $A>0$ et tout $x>1$, on a $$\ell\geq \int_1^A \frac{dt}{1+t^x}.$$
  5. En déduire que $\lim_{x\to 1^+}F(x)=+\infty$.
Indication
Corrigé
Fonctions classiques
Exercice 34 - Transformée de Fourier de la gaussienne [Signaler une erreur] [Ajouter à ma feuille d'exos]
Enoncé
On pose, pour $a>0$, $F(x)=\int_{-\infty}^{+\infty}e^{-itx}e^{-at^2}dt$.
  1. Montrer que $F$ est de classe $C^1$ sur $\mathbb R$ et vérifie, pour tout $x\in\mathbb R$, $$F'(x)=\frac{-x}{2a}F(x).$$
  2. En déduire que pour tout $x$ réel, $F(x)=F(0)e^{-x^2/4a}$, puis que $$F(x)=\sqrt\frac\pi ae^{-x^2/4a}.$$
On rappelle que $\int_{-\infty}^{+\infty}e^{-u^2}du=\sqrt \pi$.
Indication
Corrigé
Exercice 35 - Calcul de l'intégrale de Gauss [Signaler une erreur] [Ajouter à ma feuille d'exos]
Enoncé
Le but de l'exercice est de calculer la valeur de l'intégrale de Gauss $$I=\int_0^{+\infty}e^{-t^2}dt.$$ On définit deux fonctions $f,g$ sur $\mathbb R$ par les formules $$f(x)=\int_0^x e^{-t^2}dt\textrm{ et }g(x)=\int_0^{1}\frac{e^{-(t^2+1)x^2}}{t^2+1}dt.$$
  1. Prouver que, pour tout $x\in\mathbb R$, $g(x)+f^2(x)=\frac{\pi}{4}.$
  2. En déduire la valeur de $I$.
Indication
Corrigé
Exercice 36 - Calcul de l'intégrale de Gauss [Signaler une erreur] [Ajouter à ma feuille d'exos]
Enoncé
On pose $$F(x)=\int_0^{+\infty}\frac{e^{-x(1+t^2)}}{1+t^2}dt.$$
  1. Montrer que $F$ est définie et continue sur $[0,+\infty[$ et déterminer $\lim_{x\to+\infty}F(x)$.
  2. Montrer que $F$ est dérivable sur $]0,+\infty[$ et démontrer que $$F'(x)=-\frac{e^{-x}}{\sqrt x}\int_0^{+\infty}e^{-u^2}du.$$
  3. En intégrant $F'$ sur $]0,+\infty[$, montrer que $\int_0^{+\infty}e^{-t^2}dt=\frac{\sqrt \pi}2.$
Indication
Corrigé
Enoncé
Soit $f:\mathbb R\to \mathbb R$ définie par $$f(x)=\int_0^\pi \cos(x\sin\theta)d\theta.$$
  1. Montrer que $f$ est de classe $C^2$ sur $\mathbb R$.
  2. Vérifier que $f$ est solution de l'équation différentielle $$xf''(x)+f'(x)+xf(x)=0.$$
  3. Démontrer que $f$ est développable en série entière.
Indication
Corrigé
Enoncé
Pour $x\in\mathbb R$, on définit $\Gamma(x)=\int_0^{+\infty}t^{x-1}e^{-t}dt$.
  1. Quel est le domaine de définition de $\Gamma$?
    1. Pour $k\geq 1$ et $0<A<B<+\infty$, on pose $$g_k(t)=\left\{\begin{array}{ll} t^{A-1}e^{-t}|\ln t|^k&\textrm{ si }0<t<1\\ t^{B-1}e^{-t}|\ln t|^k&\textrm{ si }t\geq 1. \end{array}\right. $$ Démontrer que $g_k$ est intégrable sur $]0,+\infty[$.
    2. En déduire que $\Gamma$ est $C^\infty$ sur son domaine de définition, et calculer $\Gamma^{(k)}$.
  2. Montrer que pour tout $x>0$, $\Gamma(x+1)=x\Gamma(x)$. En déduire $\Gamma(n+1)$ pour $n$ un entier et un équivalent de $\Gamma$ en $0$.
  3. Montrer que $\Gamma$ est convexe.
    1. Justifier que, pour tout $u<-1$, $\ln(1-u)\leq -u$.
    2. Pour $x>0$, on pose $$f_n(t):=\left\{ \begin{array}{ll} t^{x-1}(1-t/n)^n&\textrm{ si }t\in]0,n[\\ 0&\textrm{ si }t\geq n. \end{array}\right.$$ Démontrer que $\lim_{n\to+\infty}\int_0^{+\infty}f_n(t)dt=\Gamma(x).$
  4. En déduire que pour $x>0$, on a $$\Gamma(x)=\lim_{n\to+\infty}n^x\int_0^1 u^{x-1}(1-u)^n du.$$
  5. En utilisant des intégrations par parties successives, conclure que, pour tout $x>0$, on a $$\Gamma(x)=\lim_{n\to+\infty}\frac{n!n^x}{x(x+1)\dots(x+n)}.$$
Indication
Corrigé
Exercice 39 - Transformée de Laplace et fonctions intégrales [Signaler une erreur] [Ajouter à ma feuille d'exos]
Enoncé
Soit $f:[0,+\infty[\to\mathbb R$. On note $I=]0,+\infty[$ et on suppose que pour tout $x\in I,$ la fonction $t\mapsto e^{-xt}f(t)$ est intégrable sur $]0,+\infty[$. On définit la transformée de Laplace $Lf:I\to\mathbb R$ par, pour tout $x\in I,$ $$Lf(x)=\int_0^{+\infty}e^{-xt}f(t)dt.$$
  1. Démontrer que $Lf$ est continue sur $I$.
  2. On suppose dans cette question que $f$ est bornée sur $\mathbb R$. Démontrer que $\lim_{x\to +\infty}xLf(x)=f(0)$ (théorème de la valeur initiale).
  3. On suppose dans cette question que $f$ admet une limite $\ell_\infty$ en $+\infty.$ Démontrer que $\lim_{x\to 0}xLf(x)=\ell_\infty$ (théorème de la valeur finale).
Indication
Corrigé
Exercice 40 - Transformée de Laplace et intégrales à paramètres [Signaler une erreur] [Ajouter à ma feuille d'exos]
Enoncé
Soit $f:\mathbb R_ +\to\mathbb C$ une fonction continue. Pour $x\in\mathbb R$, on pose $Lf(x)=\int_0^{+\infty}f(t)e^{-xt}dt.$
  1. Montrer que si $\int_0^{+\infty}f(t)e^{-xt}dt$ converge, alors $\int_0^{+\infty}f(t)e^{-yt}dt$ converge pour $y>x$.
  2. Quelle est la nature de l'ensemble de définition de $Lf$?
  3. On suppose $f$ bornée. Montrer que $\lim_{x\to+\infty}Lf(x)=0$.
Indication
Corrigé
Exercices théoriques
Enoncé
Soit $f$ une application définie sur $[0,1]$, à valeurs strictement positives, et continue. Pour $\alpha\geq 0$, on pose $F(\alpha)=\int_0^1 f^\alpha(t)dt$.
  1. Justifier que $F$ est dérivable sur $\mathbb R_+$, et calculer $F'(0)$.
  2. En déduire la valeur de $$\lim_{\alpha\to 0}\left(\int_0^1 f^{\alpha}(t)dt\right)^{1/\alpha}.$$
Indication
Corrigé
Exercice 42 - Division des fonctions régulières [Signaler une erreur] [Ajouter à ma feuille d'exos]
Enoncé
Soit $f:\mathbb R\to\mathbb R$ de classe $C^\infty$.
  1. On suppose que $f(0)=0$ et on pose, pour $x\neq 0$, $g(x)=\frac{f(x)}{x}$. Justifier que, pour $x\neq 0$, $g(x)=\int_0^1 f'(tx)dt$, et en déduire que $g$ se prolonge en une fonction de classe $C^\infty$ sur $\mathbb R$.
  2. On suppose désormais que $f(0)=f'(0)=\dots=f^{(n-1)}(0)=0$ et on pose $g(x)=\frac{f(x)}{x^n}$, $x\neq 0$. Justifier que $g$ se prolonge en une fonction de classe $C^\infty$ sur $\mathbb R$.
Indication
Corrigé
Enoncé
Soient $I$ un intervalle, $f:I\times\mathbb R\to\mathbb R$ continue et $u,v:I\to\mathbb R$ continues. Démontrer que $F : x\mapsto \int_{u(x)}^{v(x)}f(x,t)dt$ est continue sur $I$.
Indication
Corrigé
Permutation limites/intégrales et fonctions définies par une intégrale